ChaseDream
搜索
返回列表 发新帖
00:00:00

In Gandania, where the government has a monopoly on tobacco sales, the incidence of smoking-related health problems has risen steadily for the last twenty years. The health secretary recently proposed a series of laws aimed at curtailing tobacco use in Gandania. Profits from tobacco sales, however, account for ten percent of Gandania's annual revenues. Therefore, Gandania cannot afford to institute the proposed laws.

Which of the following, if true, most seriously weakens the argument?

正确答案: A

更多相关帖子

524

帖子

15

好友

4712

积分

ChaseDream

注册时间
2003-03-17
精华
8
解析
查看: 3044|回复: 3
打印 上一主题 下一主题

GWD-23-Q39

[复制链接]
跳转到指定楼层
楼主
发表于 2011-7-30 21:11:31 | 只看该作者 回帖奖励 |倒序浏览 |阅读模式
?41.: GWD-23-Q39
?In Gandania, where the government has a monopoly on tobacco sales, the incidence of smoking-related health problems has risen steadily for the last twenty years. The health secretary recently proposed a series of laws aimed at curtailing tobacco use in Gandania. Profits from tobacco sales, however, account for ten percent of Gandania’s annual revenues. Therefore, Gandania cannot afford to institute the proposed laws.
?
?Which of the following, if true, most seriously weakens the argument?
?
A.All health care in Gandania is government-funded.
B.Implementing the proposed laws is not likely to cause a significant increase in the amount of tobacco Gandania exports.
C.The percentage of revenue Gandania receives from tobacco sales has remained steady in recent years.
D.Profits from tobacco sales far surpass any other single source of revenue for the Gandanian government.
E.No government official in Gandania has ever previously proposed laws aimed at curtailing tobacco use.

请教各位,我选择C。
但是答案说是A。
收藏收藏 收藏收藏
沙发
发表于 2011-7-30 23:22:12 | 只看该作者
题干说由于烟草业的利润占这个城市总收入的10%,因而实施禁令会阻碍城市经济,所以这项法律不会被实施。
A:削弱,政府每年要花钱在市民的医疗上,由于烟草导致健康问题增多,使得政府的花销增大,所以实施禁令是必要的,可以帮助政府减少医疗支出,regardless of 烟草收入的减少。
C实际是上加强,因为烟草利润占总收入最近一直保持10%,因此实施法律会降低烟草收入,于是阻碍城市的经济发展。
板凳
 楼主| 发表于 2011-7-30 23:48:17 | 只看该作者
假设A对C错,
那么A自圆其说则可以理解为: 政府支柱health care, 与10% of revenue(收入)减少没有关系。所以weaken.
那么C, 从时态上理解为现在完成时,则表明以前几年的10% of revenue(收入) 都很稳定,一旦收入减少,那么政府确实没有办法负担施政(proposed laws) 带来的后果。 是support。

但是这种理解本来就是矛盾的。政府(gov) 与 Gandania 应该是一个概念,对吧?如果政府的收入本来就减少了,它所支持的health care 怎么会不受影响呢?

理解有误的地方,谢谢大家意见。
地板
 楼主| 发表于 2011-7-30 23:56:26 | 只看该作者
而且,因为健康问题一定能造成花销增大??
如果能做假设,是否能够假设: 政府的部分经济来源是10% of annue revenue(收入), 如果这部分钱减少了,政府则cann't afford to institute the proposed laws, 因为health care is sth the gov supports.
您需要登录后才可以回帖 登录 | 立即注册

Mark一下! 看一下! 顶楼主! 感谢分享! 快速回复:

手机版|ChaseDream|GMT+8, 2024-9-17 10:35
京公网安备11010202008513号 京ICP证101109号 京ICP备12012021号

ChaseDream 论坛

© 2003-2023 ChaseDream.com. All Rights Reserved.

返回顶部